describe some of the differences between analog and digital electronics. list at least 2 differences.

Answers

Answer 1

The difference between analog and digital electronics is the signals and the components used.

Analog vs Digital

The electronics can be classified into categories: analog and digital electronics. Some differences between both of them are as follows :

Analog electronics use continuous signals while digital electronics use discrete signals or two-state signals. Analog electronics mainly uses passive components like resistors, and capacitors but sometimes use active components. Digital electronics use active components like transistors. Analog electronics use high voltage and high current whereas digital electronics use low voltage and current.

Thus, two differences between analog and digital electronics are analog uses continuous signals while digital uses discrete signals. Then, the analog uses passive components whereas digital uses active components.

Learn more about analog and digital by clicking this link :

https://brainly.com/question/23777902

#SPJ4

Answer 2

Difference between analog and digital electronics:

Analog electronic deals with continuously varying signals. Digital electronics deals with binary signals.Analog signals are continuous, digital signals are discrete.Analog electronics tend to use passive components. Digital electronics use active components more.

Analog electronics are electronics that deal with continuous signals. They generally provide a better and more accurate representation of changes in physical phenomena, such as position, pressure, and light.

Digital electronics are electronics that deal with binary signals. The digital signals for this type of electronics are represented using binary language (0's and 1's). Their circuits are usually composed of large assemblies of logic gates.

The difference between the two are the components used and the type of signal they deal with.

Learn more about digital technology at https://brainly.com/question/4371054

#SPJ4


Related Questions

A heat engine takes in 330 J of energy from a hot reservoir and performs 22.0 J of work in each cycle. Find (a) the efficiency of the engine and (b) the energy expelled to the cold reservoir in each cycle.

Answers

a) The efficiency of the engine is 67% b) The energy expelled to the cold reservoir in each cycle is  307.89 J

Qh=330 J , W= 22.0 J

a) n= W/Qh = 22 J/330 J = 0.06675 = 0.067

b) n= W/Qh = (Qh-QL)/Qh = 1 - QL/Qh

Plugging values,

0.067  =  1 - QL/330    => QL = 307.89 J

As a ratio of useful work completed to heat produced, an engine efficiency is determined. completed the task. The power delivered at the clutch or the driveshaft is referred to as the "work done" in this context. As a result, the work produced by thermodynamic expansion is reduced by losses such as friction and other losses. Today's gasoline engines are only about 30 to 35 percent efficient, meaning that about 65 cents of every dollar you spend on gas is wasted even with routine auto maintenance like a tune-up or an oil change. According to the Second rule of thermodynamics, it is impossible for heat engines to achieve 100% thermal efficiency.

Learn more about engine here:

https://brainly.com/question/2676954

#SPJ4

what is the origin of aluminium

Answers

Answer:

Aluminium is extracted from bauxite. Bauxite is the name for the ore in which around 30 to 54% of the aluminium oxide is contained. Because of the fact that aluminium has a tendency to bind with oxygen, there is very little pure aluminium found naturally.

Explanation:

A thin circular wooden hoop of mass m and radius Rrests on a horizontal frictionless plane. A bullet, also of mass m, moving with horizontal velocity v, strikes the hoop and becomes embedded in it as shown in the figure. (a) (2) Calculate the center of mass velocity (b) (2) Calculate the angular momentum of the system about the CM. (c) (32) Calculate the angular velocity w of the hoop. (d) (4) Calculate the kinetic energy of the system, before and after collision (e) (48) Find a point of the hoop which remains at rest after collision

Answers

The velocity of the center of mass of the system is the same as the velocity of the bullet, which is v. The total angular momentum of the system about the center of mass is  Lhoop + Lbullet. he kinetic energy of the system, before and after collision Khoop + Kbullet and Khoop_after + Kbullet_after respectively. The axis of rotation, which is the center of the hoop, will remain at rest after the collision.

The center of mass of the system is at point Q, which is a distance d from point O, where:d = (mP + mO)/(2*m).The velocity of the center of mass is the derivative of the position with respect to time. Since the bullet is moving with constant velocity, the position of its center of mass is also moving with constant velocity. The position of the center of mass of the hoop is not changing, so its velocity is zero. Therefore, the velocity of the center of mass of the system is the same as the velocity of the bullet, which is v.

The angular momentum of the bullet about the center of mass is:Lbullet = Ibulletw + md*v where Ibullet is the moment of inertia of the bullet about its center of mass, and d is the distance from the center of mass to the point of contact between the bullet and the hoop. The total angular momentum of the system about the center of mass is the sum of the angular momenta of the hoop and the bullet: Ltotal = Lhoop + Lbullet

Since angular momentum is conserved, this change in angular momentum must be due to the change in the angular momentum of the hoop. Therefore, we can set the change in angular momentum of the hoop equal to Ltotal and solve for the angular velocity w:

Ltotal = Lhoop + Lbullet

Ltotal = Ihoopw + mRv + Ibulletw + mdv

Ltotal = (Ihoop + Ibullet)w + (mR + md)v

w = Ltotal/(Ihoop + Ibullet) - (mR + md)*v/(Ihoop + Ibullet).

kinetic energy is conserved, this means that the change in kinetic energy of the hoop must be equal to the change in kinetic energy of the bullet. Therefore, we can set the change in kinetic energy of the hoop equal to the change in kinetic energy of the bullet and solve for the kinetic energy of the hoop after the collision:

Khoop_after - Khoop = Kbullet - Kbullet_after

Khoop_after = Khoop + Kbullet - Kbullet_after

The kinetic energy of the system after the collision is the sum of the kinetic energies of the hoop and the bullet after the collision:

Ktotal_after = Khoop_after + Kbullet_after

To know more about momentum-

https://brainly.com/question/24030570

#SPJ4

PLS HELP NEED URGENT What is the value of g, the gravitational field strength, at the surface of the planet Venus?
Round your answer to two decimal places

Answers

Answer:

8.83 N/kg

Explanation:

To solve this problem we can use the formula:

[tex]g=G\frac{M}{r^{2} }[/tex] which describes the field strength at a certain distance from a body (where g=gravitational field strength, G=Gravitational constant, M=Mass of planet, and r=distance from center of planetary mass).

We are given the information that the radius of Venus is 6.073 x 10^6m and its mass is 4.88x10^24kg.

We are also given the gravitational constant.

Since the radius of a planet is equal to the distance between the center of the planet to its surface we can use this value for 'r'.

Therefore we now have all the information we need and so we can plug these values into the equation to solve for the gravitational field strength:

[tex]g=(6.67*10^{-11})\frac{(4.88*10^{24})}{(6.073*10^{6})^{2}}[/tex]

[tex]g=(6.67*10^{-11})\frac{(4.88*10^{24})}{3.688*10^{13}}[/tex]

[tex]g=(6.67*10^{-11})*(1.323*10^{11})[/tex]

[tex]g=8.83Nkg^{-1}[/tex] (to 2.d.p)

Unpolarized red light is incident on two identical, narrow vertical slits. The photograph at right shows the interference pattern that appears on a distant screen. a. Specify the quantity or quantities that are adding to zero at the interference minima. light waves from the two slits are adding to zero is not a sufficient answer.)b. A polarizer is placed directly in front of both slits, so that the light is vertically polarized before passing through the slits. It is observed that the intensity at each point on the screen decreases by a factor of two. How can your account for the decrease in intensity at the interference maxima?c. The polarizer is slowly rotated through 360 degree . As the polarizer is rotated, it is observed that the interference pattern does not change. What is the flaw in the reasoning? Explaind. Now imagine that one polarizer is placed in front of each slit: one polarizer with its direction of polarization vertical; the other, horizontal.Would there still be locations on the screen at which the intensity is zero? Explain why or why not.

Answers

Electromagnetic waves make up light. Electric fields can be in any direction perpendicular to the propagation of unpolarized light rays. At each point, the intensity is halved.

a). Light is an electromagnetic wave. It consists of minimally intense, pulsing electric and magnetic fields that are directed in all directions perpendicular to the propagation diffraction pattern. Electric fields of light from two slits add up to zero, therefore they likewise sum to zero in terms of light intensity.

b). In light waves (unpolarized), the electric field can be in any direction perpendicular to its propagation. Polarization entails facing the electric field in all directions except one toward zero. While polarisation, for example, in the vertical direction, additional electric field components from other directions are added to the necessary (remaining) direction.

c). Electric fields at angle θ to vertical adds Ecosθ to vertical field

before polarisation: same E in all directions

I∝E²

∝[tex]\int\limits^\pi _0[/tex]E² dθ =E² π

After polarisation;

I∝ [tex]\int\limits^\pi _0[/tex] (E cosθ )²dθ =E² π/2

Hence, Intensity is halved at every point

d). Electric field cannot be stopped by slits since it is not a mechanical thing that can be.

e). The horizontal and vertical electric field vector addition cannot be zero. Therefore, there won't be any places with 0 intensity.

Learn more about Electromagnetic waves here:

https://brainly.com/question/3101711

#SPJ4

What is a Physics teachers favourite number?

Answers

Answer:

243?

I hope this helps?

#2. The magnetic field B at all points within a circular region of radius R is uniform and directed out of the page_ That region could be a cross section inside of the long solenoid. If the magnetic field decreasing at a rate of dB/dt; what are the magnitude and direction of force on a stationary negative point charge q located at points (1) a inside of that solenoid) (2) and at point b outside of that solenoid)

Answers

The magnetic field is upward diection.

What is magnitude ?

The definition of magnitude is "distance or amount." It displays an object's size or motion in either an absolute or a relative sense. It is a means of describing the magnitude or range of something. In physics, a magnitude frequently describes a length or a size.

What is magnetic field ?

The magnetic field is the region where the force of magnetism works around a magnetic substance or a moving electric charge. a magnetic field illustration that shows how the magnetic force is distributed throughout and around a magnetic substance.

Inside the circulation region at distance r,

El = d∅/dt = s (dB / dt )

Therefore E (2π r ) = (π r2). dB/dt

Therefore E = r dB/2 dt

F = qE = qr dB/ 2 dt

At points a and b, distance from Centre is r

Therefore F = qE = qr dB/2 dt

At point c, distance r = 0

Therefore F = 0

Magnetic field is increasing. Hence, induced current in an imaginary loop passing through a and b should be anti-clockwise. Force on positive charge is in the direction of induced current. Hence force at a is towards left and force at b is upward.

Therefore, the magnetic field is upward direction.

Learn more about magnitude from the given link.

https://brainly.com/question/24468862

#SPJ4

Start by modeling the forearm as a simple shape. Draw a visual overview showing all the forces and distances. List the known information and identify what must be found. Pick an axis about which to calculate the torques and determine the torque about this point due to each force acting on the forearm.Which of the following should the forearm be modeled as?A. a particle moving with constant accelerationB. a flexible rod moving with constant accelerationC. a rigid rod moving with constant accelerationD. a particle in equillibriumD.a flexible rod in equillibriumE.a rigid rod in equillibrium

Answers

Determine the torque around this point created by each force acting on the wrist and select an axis to compute the torques about.

The forearm is the upper limb component that reaches from the shoulder to the wrist. Its bone structure is made up of the ulna and radius which are positioned laterally (medially).

The term "arm" most commonly refers to the entire upper limb appendage, although in anatomy, the phrase "forearm" is used to distinguish it from that.

As a result, the stress exerted by the bicep is 7.38 times more than the weight supported.

The formula for torque is Γ=r×F=rFsin(θ). Torque is, in additional observations, the cross product of the force vector & the distance vector, where a is the angle between r & F.

Learn more about the torque at

https://brainly.com/question/28220969?referrer=searchResults

#SPJ4

Figure 7-40 shows a cord attached to a cart that can slide along a frictionless horizontal rail aligned along an x axis. The left end of the cord is pulled over a pulley, of negligible mass and friction and at cord height h = 1.6 m, so the cart slides from x1 = 5.0 m to x2 = 1.0 m. During the move, the tension in the cord is a constant 28.0 N. What is the change in the kinetic energy of the cart during the move?

Answers

The change in the kinetic energy of the cart during the move is equal to 98.04 J

According to work energy theorem, the net work done W on an object is equal to the change in the kinetic energy of the object.

W = Kf -Ki

Here, Kf is the final kinetic energy and Ki is the initial energy

The work done on an object is,

W=Fd

Here, F is the force applied on the object and dis the displacement of the object.

Equate these two equations.

Ki-K = Fd

ΔK=Fd

The force exerted on the cart is the tension Tin the chord itself. So,

F=T

Calculate the displacement of the cart.

d = √x²+h² −√x²+h²

Substitute 5.0 m for x,, 1.0 m for x,, and 1.60 m for h.

d=√(5.0 m)² +(1.60 m)² - √(1.0 m)² +(1.60 m)²

=3.36 m

Calculate the change in kinetic energy of the cart.

ΔK = Fd

Substitute 28.0 N for F and 3.36 m for d.

ΔK = (28.0 N) (3.36 m)

=94.08 J

Learn more about Kinetic energy here:

https://brainly.com/question/26472013

#SPJ4

as an automobile approaches an observer at rest, the driver sounds her 450 hz horn. if the car moves at 45 mi/hr, find the frequency heard by the stationary observer.

Answers

The frequency heard by the stationary observer is 468.44 Hz. Frequency is a measure of the number of cycles of a periodic phenomenon that occurs in a unit of time.

Frequency is an important concept in many areas of physics and engineering, including acoustics, electronics, and telecommunications. It is also used in other fields, such as biology and medicine, to describe the periodic phenomena that occur in those fields.  It is typically measured in Hertz (Hz), which is the number of cycles per second.

The frequency of the horn as heard by the stationary observer is given by the Doppler shift formula:

frequency_heard = frequency_emitted * (speed of sound + speed of observer) / (speed of sound - speed of car)

Plugging in the values given in the problem, we get:

frequency_heard = 450 hz * (331.5 m/s + 0 m/s) / (331.5 m/s - (45 mi/hr * 1609 m/hr / 3600 s))

Converting the speed of the car to meters per second, we get:

frequency_heard = 450 hz * (331.5 m/s + 0 m/s) / (331.5 m/s - 12.5375 m/s)

Simplifying the expression and converting the result to hertz, we get:

frequency_heard = 450 hz * (331.5 m/s) / (319 m/s)

frequency_heard = (450 hz * 331.5 m/s) / 319 m/s

frequency_heard = 450 hz * 1.0398936170212766

frequency_heard = 468.4414893617021 hz

So the frequency heard by the stationary observer is approximately 468.44 Hz.

Learn more about frequency, here https://brainly.com/question/29582729

#SPJ4

A ball dropped on a surface from a 2-m height bounces to a height of 0.98 m.
What is the coefficient of restitution between ball and surface?

Answers

The coefficient of restitution between the ball and the surface is equal 0.70.

What is the coefficient of restitution?

The coefficient of restitution can be defined as the ratio of the final to the initial speed between two objects after they collide. It generally ranges from 0 to 1 where 1 will be a perfectly elastic collision.

The velocity of the ball when drooped from height 'h' is given by:

[tex]v = \sqrt{2gh}[/tex]

Before impact, the velocity is given by: [tex]u_1 = \sqrt{2gh}[/tex]

[tex]\displaystyle e =\sqrt{\frac{2gh_1}{2gh}}[/tex]

[tex]\displaystyle e =\sqrt{\frac{h_1}{h}}[/tex]

Given, h = 2 m , h₁ = 0.98 m

[tex]\displaystyle e =\sqrt{\frac{0.98}{2} }[/tex]

e = 0.70

Therefore, the coefficient of restitution between the ball and the ground is 0.70.

Learn more about coefficient of restitution, here:

https://brainly.com/question/28549781

#SPJ1

a quantum oscillator in the wall of a black body cabity has frequency of 5.0*10(14) Hz. calculate the energy of the first two quantum states.

Answers

The energy of the first two quantum states, given that the black body cabity has frequency of 5.0×10¹⁴ Hz is 3.313×10⁻¹⁹ J

How do I determine the energy?

Energy is defined as the capacity to do work. However, energy and frequency are related according to the formula given below:

E = hf

Where

E is the energyh is the Planck's constant f is the frequency

We can obtain the energy of the two quantum states as shown below:

Frequency (f) = 5.0×10¹⁴ HzPlanck's constant (h) = 6.626×10⁻³⁴ JsEnergy (E) =? 6.23×10⁻¹⁹ J

Energy (E) = Planck's constant (h) × frequency (f)

E = hf

E = 6.626×10⁻³⁴ × 5.0×10¹⁴

E = 3.313×10⁻¹⁹ J

Thus, we can conclude from the above calculation that the energy is 3.313×10⁻¹⁹ J

Learn more about energy:

https://brainly.com/question/8623863

#SPJ1

Consider the photoelectric effect when one decreases only the frequency of the incoming light onto the emitter, one measures an increased current. either a. or c. You cannot determine which from the information given. no change in either current or stopping voltage. a decrease in the necessary stopping voltage. an increase in the necessary stopping voltage.

Answers

An increase in the required stopping voltage if the photoelectric effect simply affects the frequency of a incident sunlight onto to the emitter.

What is the way Einstein described the photoelectric effect?

Since photons are the units of light, according to Einstein's theory, an electron receives the entire photon's energy when it collides with a metal surface. The ejected electron receives the remaining energy as kinetic energy after using some of this energy to free it from the metal atom's hold.

What are the main points of the photoelectric effect?

Three key properties of the photoelectric effect that are not consistent with classical physics include: (1) the lack of a lag, (2) the independence of photoelectrons' kinetic energy from the strength the incident radiation, and (3) the existence of a cut-off frequency.

To know more about photoelectric effect visit:

https://brainly.com/question/26465043

#SPJ4

Patrick walks 70.0 m south in 150.0 seconds. He then walks 20.0 meters north in 30.0 seconds. What is his average velocity for the trip?

0.200 m/s north
0.500 m/s south
0.278 m/s south
0.417 m/s north

Answers

Answer:

0.500 m/s south

Explanation:

To find Patrick's average velocity for the trip, you need to find the total distance traveled and the total time taken for the trip.

Patrick walks 70.0 meters south in 150.0 seconds, for a total distance traveled of 70.0 meters. He then walks 20.0 meters north in 30.0 seconds, for a total distance traveled of 20.0 meters. The total distance traveled for the trip is therefore 70.0 + 20.0 = 90.0 meters.

The total time taken for the trip is 150.0 seconds + 30.0 seconds = 180.0 seconds.

The average velocity for the trip is the total distance traveled divided by the total time taken, so the average velocity is 90.0 meters / 180.0 seconds = 0.500 m/s.

Therefore, the correct answer is 0.500 m/s south.

an 18 kg shopping cart moving at a velocity of 0.7 m/s collides with a store wall and stops. the momentum of the shopping cart

Answers

The momentum of the cart of mass 18 kg is 12.6 kgm/s.

What is momentum?

Momentum is the product of mass and velocity.

To calculate the momentum of the shopping cart, we use the formula below.

Formula:

M = mv........................ Equation 1

Where:

M = Momentumm = Mass of the shopping cartv = Velocity of the shopping cart

From the question,

Given:

m = 18 kgv = 0.7 m/s

Substitute these values into equation 1

M = 18×0.7M = 12.6 kgm/s

Hence, the momentum of the cart is 12.6 kgm/s.

Learn more about momentum here: https://brainly.com/question/22257327

#SPJ1

The momentum of the shopping cart will be 12.6Kgm/s.

What is Momentum?

Momentum is defined as the product of mass of an object and velocity of that object. It is vector quantity and it has both direction and magnitude.

The formula for momentum is written as;

p = m × v

p = momentum

m = mass of the object

v = velocity

Types of MomentumLinear MomentumAngular Momentum

p = m × v

where m = 18kg

v = 0.7m/s

p = 18 x 0.7

p = 12.6 Kgm/s

Learn more about momentum here: https://brainly.com/question/18798405

#SPJ1

Plaskett's binary system consists of two stars that revolve In a circular orbit about a center of mass midway between them. This statement implies that the masses of the two stars are equal (see figure below). Assume the orbital speed of each star is |v | = 240 km/s and the orbital period of each is 12.5 days. Find the mass M of each star. (For comparison, the mass of our Sun is 1.99 times 1030 kg Your answer cannot be understood or graded. More Information solar masses

Answers

Each star has a mass M of 1.43 × 10³²kg.

The simplest types of orbits in celestial mechanics are circular orbits, in which an orbiting body moves around a gravitational mass while maintaining a constant radius.

Calculation:We learn from the question that the stars'

masses are m1 = m2 = M.

Each star orbits at a speed of Vs = 240 km/s or 240000 m/s.

T = 12.5 days, often known as 12.5246060, or 1080000sec, is the orbital period.

The mathematical formula for the centripetal force exerted on these stars is

Fc = Mv²/r

The mathematical formula for the gravitational force exerted on these stars is

Fg = GM²/d²

so Fc = Fg

Mv²/r = Gm₁ₓm₂/d²

v²/r = GM/(2r)²

v²/r = GM/4r²

M = v² × 4r /G

Each sun travels a certain distance during one cycle, which is mathematically expressed as

D = V × T

D = 240000 × 1080000

D = 2.592×10¹¹ m

Now this can also be represented as

D = 2πr

Therefore

2πr =  2.592×10¹¹ m

r = 2.592×10¹¹ / 2π

r = 4.124 × 10¹⁰ m

So,

M = v² × 4r/ G

M = (240000)² × 4 ×  4.124 × 10¹⁰/ 6.67 × 10 ⁻¹¹

M =  1.43 × 10³²kg.

To know more about circular orbits visit:-

https://brainly.com/question/29679142

#SPJ4

Complete the given reaction which is part of the electron transport chain. The abbreviation abbreviation for the product. represents coenzyme Q. Use this FADH,+Q- The reactant that is reduced is In complex III, electrons are transferred from coenzyme Q to cytochrome C, which contains iron. QH; +2 cyt (Fe) —Q+2 cyt (Ft) +2H+ Determine the oxidation number for iron on the right side of the reaction arrow

Answers

Make use of this FADH,+Q- The reduced reactant is In complex III, iron-containing cytochrome C receives electrons from coenzyme Q. QH; +2 cyt (Fe); Q+2 cyt (Ft); +2H+. Iron's oxidation number is +2, which is shown on the right side of the reaction arrow.

FADH₂ + Q  [tex]\rightarrow[/tex]  QH2+ FAD+

Coenzyme Q is reduced in this situation, and FADH2 is oxidized, hence the correct response is Q.

reduced Complex III oxidizes coenzyme Q, cut chromosome c is reduced, resulting in the reduction of Fe3+ to Fe2+, and the iron on the right side is now in the +2 oxidation state.

so the answer is x=+2

Subatomic particles known as electrons have a -1 magnitude elementary charge. The magnitude of an electron's charge is equivalent to that of a proton (but has an opposite sign).

Because of this, electrically neutral atoms and molecules need to contain an equal number of protons and electrons. The size and mass of an electron are significantly smaller than those of a proton, despite the fact that the magnitude of the charges borne by protons and electrons is the same (the mass of an electron is around 1/1837 that of a proton).

Protons, electrons, and neutrons are the fundamental subatomic particles that were later identified by scientists. At the core of each atom is a large mass known as the nucleus. Both neutrons and protons are found in the nucleus. Take into account the solar system.

Learn more about electrons here:

https://brainly.com/question/1255220

#SPJ4

Calculate the height (in m) of a cliff if it takes 2.17 s for a rock to hit the ground when it is thrown straight up from the cliff with an initial velocity of 8.16 m/s.


How long (in s) would it take to reach the ground if it is thrown straight down with the same speed?

Answers

The height of the cliff will be equal to 5.37 meters, and the time taken to reach the ground if it is thrown straight down with the same speed will be equal to 0.50 s.

What is Speed?

The amount of the shift in approach per unit of time or the size of the displacement over time for an object can be used to describe speed, which would be a scalar quantity in everyday language and kinematics.

The maximum speed that can be maintained when a time period grows closer to zero is the starting speed.

As per the given information in the question,

Use the equation given below to find the vertical height of the cliff.

y = y₀ + ut + 1/2 at²

Here,

y represents final position of the object

y₀ is the initial position of the object and t is the time interval.

So,

y₀ = y -ut -1/2at²

y₀ = 0 - (8.16)(2.17) - 1/2(-9.8)(2.17)²

= -17.70 + 23.07

= 5.37 meters.

(b)

Now again rearrange the equation,

y = y₀ + ut + 1/2at² for y - y₀

Now, find the quadratic equation in time,

t = -u ± √[u² - 4(1/2a)(y₀ - y)]/2(1/2a)

Put the values in the above equation,

= -(-8.16 m/s) ± √[(-8.16)² - (-9.8)(2)(5.37)]

= [8.16 ± √(66.58 + 105.25)]/-9.8

= [8.16 ± 13.1]/-9.8

= -4.94/-9.8 = 0.50 s.

To know more about Speed:

https://brainly.com/question/28224010

#SPJ1

Researchers interested in identifying the optimal planting density for a type of perennial grass performed the following randomized experiment: Ten different plots of land were each divided into eight subplots, and planting densities of 2, 4, 6 and 8 plants per square meter were randomly assigned to the subplots, so that there are two subplots at each density in each plot. At the end of the growing season the amount of plant matter yield was recorded in metric tons per hectare. These data appear in the file pdensity. dat. The researchers want to fit a model like y = β1 + β2x + β3x 2 + , where y is yield and x is planting density, but worry that since soil conditions vary across plots they should allow for some across-plot heterogeneity in this relationship. To accommodate this possibility we will analyze these data using the hierarchical linear model described in Section 11.1. Randomized block design: Researchers interested in identifying the optimal planting density for a type of perennial grass performed the following randomized experiment: Ten different plots of land were each divided into eight subplots, and planting densities of 2, 4, 6 and 8 plants per square meter were randomly assigned to the subplots, so that there are two subplots at each density in each plot. At the end of the growing season the

Answers

I answered it in general way

Since the field is divided into subplots and a random sample is selected from each subplot, stratified sampling is used.

Sampling is the procedure used by researchers to choose a subset of participants from a larger group, given that it is usually impossible to evaluate the entire population. In the case of stratified sampling, this entails breaking the overall population into smaller groups, known as strata, and then choosing a particular number of people from each strata. This approach ensures that the sample is chosen at random, ensuring the objectivity of the research. The approach used in the example provided was stratified sampling since a 46-acre field's general population was separated into subplots that represent was split up into smaller plots that stand in for the strata. The researcher then chose one random sample following that.

To know more about random sample

https://brainly.com/question/16805790

#SPJ4

gfcis operate by comparing the amount of current flowing in the circuit conductors of a 240-volt circuit. when these are not the same, the gfci device trips.

Answers

A GFCI measures the amount of current that flows from hot to neutral.

What is circuit ?

A circuit is a closed channel in electronics that permits electricity to flow from one place to another. It may contain a variety of electrical components such as transistors, resistors, and capacitors, but the flow is not hampered by a gap or break in the circuit.

A simple circuit is illustrated using a flashlight. When the switch is turned off, the circuit is not closed, which means that no electrical current flows from the batteries to the flashlight's bulb. When you turn the switch on, a piece of metal in the flashlight physically plugs the circuit gap. Electricity goes from the batteries to the bulb, causing it to light up.

A GFCI measures the amount of current that flows from hot to neutral. If there is an imbalance, the circuit will trip. The GFCI detects mismatches as tiny as 4 or 5 milliamps and can respond in as little as one-thirtieth of a second.

To learn more about circuit follow the given link: https://brainly.com/question/11798838

#SPJ1

one of the features that is helpful in identifying many metamorphic rocks is the presence of foliation. foliation is the apparent layering within a rock due to the alignment of minerals from compression or shear stress. below are three dough balls experiencing stress. select the one that is experiencing shear stress.

Answers

Based on the provided figures, the dough balls which is experiencing shear stress is option C.

Shear stress refers to force tending to cause deformation of a material by slippage along a plane or planes parallel to the imposed stress. The resultant shear is significant in nature, being intimately related to the downslope movement of earth materials and to earthquakes. Shear stress may also happen in solids or liquids. Shear stress causes foliation. When a rock is acted upon by pressure that is not the same in all directions, or by shear stress, minerals can become elongated in the direction perpendicular to the main stress and the pattern of aligned crystals that results is known as foliation. Foliation is among the features that are helpful in identifying many metamorphic rocks.

Learn more about Shear stress:

https://brainly.com/question/18485445

#SPJ4

Particle a is pursuing particle b along the x-axis of a frame S. The two masses are ma and mo (with va > vb). When a catches up with b, they collide and coalesce to form a single particle of mass m and speed v. (a) (10 points) Find m and v. (b) (5 points) How much kinetic energy is lost in this collision? Your answers should be in terms of ma, mb, va and vb.

Answers

A particle of mass m is travelling with an initial velocity of u I along the x-axis.

It moves with half of its initial kinetic energy after elasto-magnetically colliding with a particle of mass 10 m at rest (see figure). If sin 1 = n sin 2, then n has value. A particle of mass m collides with a particle of mass 2m that is initially at rest while travelling along the x-axis at a speed of v. Following the collisions, as shown in the picture, the second particle has split into two equal-mass pieces that travel at equal angles theta? 0 with the x-axis. The first particle has come to rest. The x-axis is being traveled by a particle. Its speed (v) along with.

To learn more about velocity please click on below link

https://brainly.com/question/18084516

#SPJ4

(Come) back to the future. Suppose that a father is 21.00 y older than his daughter. He wantsto travel outward from Earth for 2.000 y and then back to Earth for another 2.000 y(both intervals as he measures them) such that he is then 21.00 y younger than hisdaughter. In terms of c, what constant speed (relative toEarth) is required for the trip?
1_________ c

Answers

The constant speed (relative to earth) required for the trip is 0.9959.

Solution:

Then these three equations combine to give a single condition from which γ can be determined (and consequently v ):

44 = 4y = y=11 = β = 2√3/11  = 0.9959

A car moving at a constant speed is an example of constant speed because it travels the same distance in the same amount of time. Aircraft systems use constant-speed drives to transfer power to an onboard generator at a constant output speed regardless of engine input speed.

Even if the speed is constant the speed will change if you are moving in a circular motion. This is because when an object moves in a circle, the direction of movement is always changing. Therefore the speed is not constant. DC motors have a large installed base and provide maximum torque at zero speed.

Learn more about Constant speed here:- https://brainly.com/question/2681210

#SPJ4

Can someone please help with this physics problem.

Answers

Normal force that will act on the body moving downward will be  480 newtons.

What is Newton's second law?

One of the most significant laws in all of physics is Newton's second law. F = ma, wherein F (force) as well as a (acceleration) both are vector values, can be used to represent an object with mass m is constant. A body accelerates according to the equation if there is a net force that acts on it.

what is Newton's first law?

Until and unless an external force occurs on a body, it is in a condition of rest or homogeneous movement in a straight line. According to Newton's first law of motion, a body won't begin to move unless and until an outside force impacts it.

Briefing:

given:

m=20 kg, a= 2.0m/s^2, g=10 m/s^2

   Mg-F(normal)=Ma

  60*10-F(normal)=60*2

 F(normal)=480N

To know more about Force visit:

https://brainly.com/question/13191643

#SPJ1

 

a 355 ml soda can is 6.2 cm in diameter and has a mass of 20 g. such a soda can half full of water is floating upright in water. What length of the can is above the water level?

Answers

The length of the can above the water level is  0.052195 m  .

In the question ,

it is given that ,

capacity of the soda can = 355 ml = 0.000355 m³

diameter of the soda can = 6.2 cm = 0.062 m

mass of the soda can = 20g = 0.02 Kg

given that the soda can is half filled , so , the water volume in the can is

= 0.000355/2 m³

= 0.0001775 m³

Let the density of the water be  = 1000 kg/m³

So , the mass of the half filled water will be = 1000×0.0001775

Let water density be 1000kg/m³ , the mass of this half-filled water is

= 0.1775

So , The total mass of the water-can system is = 0.1775 + 0.02 = 0.1975 kg

Now , For the system to stay balanced, the total mass should be equal to the mass of water displaced by can submerged

So , The volume of water displaced by submerged can is

= 0.1975/1000 = 0.0001975 m³

So , Volume of the can that is not submerged( above water level) is

= 0.000355 - 0.0001975

= 0.0001575 m³

We know that base area of  can = πr² = π(d/2)² = π×(0.062/2)²

= 0.00301754 m²

So , the length of the can above water is = 0.0001575/0.00301754

= 0.0521948342 m

≈ 0.052195 m

Therefore , the can will be 0.052195 m above the water level .

Learn more about Length here

https://brainly.com/question/14804736

#SPJ4

A given net force propels an object along a straight-line path. If the mass were doubled, its acceleration would O quadruple be half double stay the same

Answers

A given net force propels an object along a straight-line path. If the mass were doubled, its acceleration would be half.

What is the relationship between force and acceleration mass?

Force is mass times acceleration, or F = m * a. This means an object with a larger mass requires a stronger force to be moved along at the same acceleration as an object with a small mass. This is Newton's Second Law of Motion.

Newton's Second Law of Motion defines the relationship between force and acceleration. They are directly proportional. If you raise the force applied to an object, the acceleration of that object raises by the same factor. In short, force equals mass times acceleration.

Learn more about Newton's Second Law at: https://brainly.com/question/13447525

#SPJ4

A hiker who weighs 985 N is strolling through the woods and crosses a small horizontal bridge. The bridge is uniform, weighs 3610 N, and rests on two concrete supports one at each end. He stops one-fifth of the way along the bridge. What is the magnitude of the force that a concrete support exerts on the bridge (a) at the near end and (b) at the far end?

Answers

The magnitude of the force that a concrete support exerts on the bridge (a) at the near end is 2593 N  (b) at the far end is  2002 N    .

In the question ,

it is given that ,

the hiker weighs : [tex]W_{h}[/tex] = 985 N

the bridge's weight ([tex]W_{b}[/tex]) is = 3610 N ,

let the magnitude of the force that concrete support exerts on bridge at near end is = F₁    and

let magnitude of force that concrete support exerts on bridge at far end is = F₂ .

Since the bridge is in equilibrium   , the sum of all the forces in the vertical direction will be = 0 .

So , F₁ + F₂ -  [tex]W_{h}[/tex] - [tex]W_{b}[/tex] = 0

Substituting the value ,

we get ,

F₁ + F₂ -  985 - 3610 = 0

F₁ + F₂ = 4595 N

also given that , the hiker is at 1/5th position of the length of bridge from left support = L/5 m .

Since the bridge is not rotating , the net torque will be 0 .

[tex]T_{net}[/tex] = 0 .

The torque due to weight of hiker, due to weight of bridge and due to right end support about point nearest support is written as :

So ,  [tex]W_{h}[/tex]×(L/5) +  [tex]W_{b}[/tex]×(L/2) - F₂×L = 0

[tex]W_{h}[/tex]×(1/5) +  [tex]W_{b}[/tex]×(1/2) = F₂

985×(1/5) + 3610×(1/2) = F₂

So , F₂ = 2002 N

Substituting the value of force in the equation F₁ + F₂ = 4595 N  ,

we get ,

F₁ + 2002 = 4595

F₁ = 4595 - 2002

F₁ = 2593 N

Therefore , The magnitude of the force at the near end is 2593 N and at the far end is 2002 N .

Learn more about Force here

https://brainly.com/question/14282350

#SPJ4

an object with height 0.85 cm is placed a distance 19.75 cm in front of a convex mirror with focal length -6.25 cm,removedde6d0d485ec413711a4e98afe87284678ac0bba70217810ba692cdd66aafdf8cremoved 2m68-57-20-4f-a59d-29897 20% part (a) calculate and enter a value for the magnitude of the distance between the image and the mirror given the values in problem statement. |di||di|

Answers

The distance between the image and the mirror is -4.63 cm.

The mirror equation is given by,

1/u + 1/v = 1/f

where, u is the distance to the object = 17.95 cm

v is the distance to the image

f is the focal length = -6.25 cm

Placing the values in the equation, we have

1/17.95 + 1/v = -1/6.25

1/v = -1/6.25 - 1/17.95

1/v = - 0.16 - 0.056

1/v = - 0.216

v = -4.63 cm ( minus sign indicates that the image is behind the mirror and it is virtual)

Thus, the image distance is -4.63 cm.

To know more about mirrors:

https://brainly.com/question/29195164

#SPJ4

Listed following are some of the distinguishing characteristics of the four Galilean moons of Jupiter. Match each characteristic to the appropriate moon: lo
source of ionized gas in
the donut-shaped charged
particle belt around Jupiter
hot, glowing lava
visible in some photos
volcanoes currently
erupting

Answers

Volcanoes that are currently erupting are the source of the ionized gas in the charged particle belt that resembles a donut around Jupiter.

Any particle with an electric charge is referred to as a charged particle in physics. It could be an ion, such as a molecule or atom having an excess or shortage of electrons in comparison to protons. Another possibility is that it's an elementary particle like an electron, proton, or another one with the same charge (except antimatter). Atomic nuclei without electrons, like alpha particles, could also be charged particles.

The term "plasma" refers to a mixture of charged particles, atomic nuclei, and separated electrons, but it can also refer to a gas with a considerable amount of charged particles.

Positive (+) and negative (-) charges are randomly assigned to charges (-). Positive charges don't have any intrinsic properties; the only known "types" of charges are the two that are currently known.

Learn more about charged particle here:

https://brainly.com/question/11158712

#SPJ4

Mary weighs 525 N and she walks down a flight of stairs to a level 6.5 m below her starting point. What is the change in Mary’s potential energy? Answer in units of J.

Answers

The change in Mary’s potential energy is - 3412.5 Joule.

What is gravitational potential energy?

The energy that an item has or acquires when its location changes as a result of being in a gravitational field is known as gravitational potential energy. Gravitational potential energy can be defined as an energy that has a connection to gravitational force or gravity.

Given that:

Weight of Marry: W = 525 N.

She walks down a flight of stairs to a level 6.5 m.

Hence, decrease in potential energy of her = Wh

= 525 N × 6.5 m

= 3412.5 Joule.

Hence, the  decrease in potential energy of her is 3412.5 Joule.

Learn more about gravitational potential energy here:

https://brainly.com/question/19768887

#SPJ1

Other Questions
Anna is knitting hats to give as gifts. Each one will use 1.2 skeins, and she has 6 skeins of yarn available. With the yarn available, how many hats can Anna knit? How did states discriminate against nonwhite voters? select three options. by sponsoring voter registration drives by charging voters taxes before they could vote by allowing only white voters in primary elections by asking african americans to vote only in primaries by intimidating african american voters at the polls by organizing peaceful sit-ins and marches A water balloon is thrown upward from a height of 5 feet with an initial velocity of 35 feet per second. The quadratic function \large h\left(t\right)=-16t^2+35t+5 represents the height of the balloon, h, in feet t seconds after it is thrown. When does the water balloon reach the height of 20 feet? round your answer to the nearest thousandth. which answer correctly names the notes on the staff from left to right? question 12 options: a g f f e d d c b c b a After acquiring three new rental properties, Savannah made improvements to them. In the first property, she installed ceiling fans in each bedroom. In the second, she laid out decorative throw rugs in the living room. In the third, she had a new central air unit installed. Six months later, she decides to sell the properties and is wondering which of these improvements she can remove from the properties and keep for herself. Of the three improvements, Savannah CANNOT remove: a. all three improvements because they would be considered fixtures that become part of the real property b. the ceiling fans and central air unit, but their classification as fixtures would make no difference in whether or not Savannah can remove them c. the ceiling fans and central air unit because they would be considered fixtures d. the central air unit and the throw rugs because they would be considered fixtures What was the Santa Fe Indian School like in its early years? What is it like when Michelle Obama speaks there in 2016? Use details from Obamas speech as evidence. explain why stainless steel is more corrosion resistant in many environments than plain carbon steel. A wire of negligible mass is wrapped around the outer surface of the disk of mass M. If the disk is released from rest, determine its angular velocity at time t. Given: M = 2 kg , t = 3 s , r = 80 mm When troubleshooting network issues, it's important to carry out tasks in a specific order. Drag the trouble shooting task on the left to the correct step on the right. Drag Drop Step 1 Establish a plan of action. Identify he problem. Implement the solution or escalate. Step 2 Test the theory to determine the cause. Establish a theory of probable cause. Step 3 Step 4 Document findings, actions, and outcomes. Verify full system functionality. Step 5 Step 6 Step 7 How far east did the Germans advance in World War I? I need conclusions please and thank youI will really appreciate it What is theme of this fable? the smallest among us has the greatest role when All of are following are same process. EXCEPT A.CondensationB.FreezingC.Crystallization D.Solidification Psychological knowledge is advanced through a process known as ________, which involves a prescribed series of steps designed to achieve the desired knowledge.O deductive reasoningO the scientific methodO the experimentO inductive reasoning rna polymerase from e. coli (core enzyme alone) has all of the following properties except that it: light travels at about 300 million meters per second, and the sun is about 150 million kilometers from the earth. how long does it take light from the sun to reach earth? 17. A skateboarder is going 10 m/s the instant it is launched vertically off a ramp. If the mass of the skateboarder is 50 kg, how high above the ramp can he possibly go? If the mass of the skateboarder is 100 kg, how high above the ramp can he possibly go? If the speed of the skateboard is 20 m/s when it is launched how high above the ramp can he possibly go? (5.1m, 5.1m, 20.4m) What information does a molecular formula provide?A. the number and kind of atoms that are bonded by the transfer of electronsB. the simplest whole number ration of atoms that are bonded by the transfer of electronsC. information about a molecule's structureD. the number and kind of electrons present in a molecule jessica isn't invited to a super bowl party her coworkers are throwing because she's a woman. jessica is experiencing from here coworkers. What are the vertex and range of y = |2x + 6| + 2? 1 (0, 2); 2 < y < 2 (0, 2); y < 3 (3, 2); 2 < y < 4 (3, 2); y <